Rozkład osobliwy

Procesy stochastyczne. Sposoby racjonalizowania wielkich ilości informacji. Matematyka w naukach społecznych.
ika0906
Użytkownik
Użytkownik
Posty: 6
Rejestracja: 16 kwie 2015, o 19:46
Płeć: Kobieta
Lokalizacja: Katowice

Rozkład osobliwy

Post autor: ika0906 »

Witam! :)
Mam problem ze zrozumieniem rozwiązania pewnego zadania. Wiem, ze ono się już gdzieś tutaj pojawiło, ale chciałabym prosić o wyjaśnienie kilku kwestii rozwiązania, które ja mam. Pytania zamieszczałam na bieżąco.
A oto jego treść i rozwiązanie:
Niech \(\displaystyle{ ( \Omega, F, Pr)}\) będzie przestrzenią probabilistyczną oraz niech \(\displaystyle{ X_{1}, X_{2}, \cdots, X_{n}, \cdots}\) będą niezależnymi zmiennymi losowymi o tym samym rozkładzie \(\displaystyle{ Pr(\omega;X_{i}(\omega) = 1) = p}\) i \(\displaystyle{ Pr(\omega; X_{i}(\omega)=0)=1-p}\) dla \(\displaystyle{ i \in \{1, 2, \cdots, n\}}\). Przez \(\displaystyle{ X}\) ze zbioru \(\displaystyle{ \Omega}\) o wartościach rzeczywistych oznaczmy zmienną losową daną wzorem \(\displaystyle{ X(\omega) =\sum_{k=1}^{ \infty}\frac{X_{k}(\omega)}{2^k}}\). Pokaż, że jeśli \(\displaystyle{ p \neq \frac{1}{2}}\), to zmienna losowa \(\displaystyle{ X}\) ma rozkład osobliwy.

ROZWIĄZANIE:
( Z góry przepraszam za wszystkie błędy )

\(\displaystyle{ \textbf{Rozkład osobliwy}}\) Zmienna losowa \(\displaystyle{ X}\) ma rozkład singularny (osobliwy), jeśli jej dystrybuanta jest funkcją ciągłą i istnieje taki zbiór \(\displaystyle{ A \subset \mathbb{R}}\), że \(\displaystyle{ l_{1}(A) = 0}\), ale \(\displaystyle{ P(X \in A) = 1}\).

\(\displaystyle{ \textbf{I. Ciągłość dystrybuanty}}\)

Z definicji dystrybuanty: \(\displaystyle{ F_{X}(x) = P(X \leq x)}\) oraz z własności dystrybuanty dla liczb rzeczywistych na prostej mamy, że \(\displaystyle{ F_{X}(x) - F_{X}(x^{-}) = P(X = x).}\)

Mamy pokazać, że \(\displaystyle{ P(X = x) = 0}\).

Ustalmy \(\displaystyle{ x \in [0,1]}\) oraz \(\displaystyle{ X = \sum_{k=1}^{ \infty}\frac{ \delta_{k}}{2^k}, \delta_{k} \in \{0,1\}}\) dla \(\displaystyle{ k \in \{0,1, \cdots \}}\).

Wiemy, że każda liczba rzeczywista z przedziału \(\displaystyle{ [0,1]}\) ma co najwyżej dwa różne przedstawienia w systemie dwojkowym.

Zatem jesli \(\displaystyle{ x}\) jest niewymierne dwójkowo ( czyli ma nieskończone rozwinięcie dwójkowe), to \(\displaystyle{ \{X = x \} = \bigcap_{k=1}^{\infty}{\{x_{k} = \delta_{k}\}}}\) ( Dlaczego tak jest? Skąd to wynika? )
Dalej,
\(\displaystyle{ A_{n} =\bigcap_{k=1}^{n}{\{x_{k} = \delta_{k}\}}}\) jest ciągiem zdarzeń ( Skąd to wynika? Po prostu sobie ustalamy taki skończony ciąg zdarzeń?)

\(\displaystyle{ \{x_{k} = \delta_{k}\} = \{0,1\} \times \{0,1\} \times \cdots \times \{\delta_{k}\} \times \{0,1\} \times \cdots}\) (Dlaczego tak mozemy to zapisać? Kojarzy mi się to z miarą produktywną, ale nie potrafię tego wytłumaczyć dlaczego mamy taki zapis )

Ponadto, \(\displaystyle{ A_{n}}\) jest ciągiem zstępującym, czyli \(\displaystyle{ A_{n+1} \subset A_{n}}\) dla \(\displaystyle{ n = 1, 2, \cdots}\) ( Dlaczego jest zstępujący? Skąd to wynika?)

Dalej mamy \(\displaystyle{ A_{n} = \{\delta_{n}\} \times \{\delta_{n}\} \times \cdots \times \{\delta_{n}\} \times \{0,1\} \times \cdots}\) ( Dlaczego tak zapisujemy?)

W takim razie \(\displaystyle{ P(\{X=x\}) = P(\bigcap_{n=1}^{\infty}{A_{n}}) = \lim_{n \to \infty} P(A_{n}) = 0}\) ( Dlaczego ta granica jest równa zero? Skąd to wynika? Można to jeszcze rozpisać ?)

I zasadnicze pytanie: Gdzie wykorzystujemy albo jak wykorzystać to że \(\displaystyle{ p \neq \frac{1}{2}}\) ? Mam jeszcze pod tą częścią napisane coś takiego : \(\displaystyle{ P(A_{n}) = p^{r}(1-p)^{n-r} \leq a_{n}}\), gdzie \(\displaystyle{ 0 \leq r \leq n}\) oraz \(\displaystyle{ 1-p , p \leq a < 1}\). O co tutaj w ogóle chodzi ? Czy tutaj właśnie korzystamy z tego, że to \(\displaystyle{ p \neq \frac{1}{2}}\)?

II. Istnienie zbioru \(\displaystyle{ A \subset \mathbb{R}}\) miary Lebesgue'a zero, takiego że \(\displaystyle{ P(X \in A) = 1}\).

\(\displaystyle{ \textbf{Liczba normalna}}\) Liczbę \(\displaystyle{ a \in [0,1]}\) nazywamy normalną przy podstawie \(\displaystyle{ d}\), gdzie \(\displaystyle{ d \in \mathbb{N}, d \geqslant 2}\), jeżeli ma ona przedstawienie \(\displaystyle{ \begin{displaymath} a =\sum_{n=1}^{ \infty}\frac{ \varepsilon_{n}}{d^n}, \ \ \ 0 \leq \varepsilon_{n} \leq d-1 \end{displaymath}}\)
spełniające warunek
\(\displaystyle{ \begin{displaymath}
\lim_{n \to \infty} \frac{card \{i: \varepsilon_{i} = k, i = 1, 2, \cdots, n\}}{n}
= \frac{1}{d}, \ \ \ k \in \{0, 1, \cdots , d-1\}
\end{displaymath}}\)


\(\displaystyle{ \textbf{Twierdzenie Borela }}\) Zbiór wszystkich liczb normalnych przy podstawie \(\displaystyle{ 2}\) ma miarę Lebesgue'a równą \(\displaystyle{ 1}\). ( Inaczej, prawie wszystkie ( w sensie miary Lebesgue'a) liczby z przedziału \(\displaystyle{ [0,1]}\) są normalne względem każdej podstawy).

Zatem oznaczmy \(\displaystyle{ S = X_{1} + X_{2} + \cdots + X_{n}}\) oraz \(\displaystyle{ S_{n} = \sum_{k=1}^{ n}{ X_{k}}}\) (Czy to oznaczenie jest poprawne?)

W takim razie z Mocnego Prawa Wielkich Liczb mamy \(\displaystyle{ \begin{displaymath}
P(\lim_{n \to \infty}
\frac{S_{n}}{n}
= p) = 1,
\end{displaymath}}\)
gdzie \(\displaystyle{ p \neq \frac{1}{2}}\)

Gdyby \(\displaystyle{ p = \frac{1}{2}}\), to \(\displaystyle{ P(\lim_{n \to \infty}
\frac{S_{n}}{n}
\neq p) = 1}\)
, ale \(\displaystyle{ \begin{displaymath} P(\lim_{n \to \infty}
\frac{card \{i: X_{i} = k, i = 1, 2, \cdots, n\}}{n}
= \frac{1}{2}) = 1, \ \ \ k \in \{0, 1\}\end{displaymath}}\)
(z def. liczb normalnych i MPWL i daje sprzeczność z powyższym, gdy \(\displaystyle{ p= \frac{1}{2}}\)).
Zatem niech \(\displaystyle{ A}\) będzie zbiorem liczb normalnych przy podstawie \(\displaystyle{ 2}\). Wtedy, z powyższych rozważań wynika, że X przyjmuje wartości z dopełnienia zbioru liczb normalnych przy podstawie \(\displaystyle{ 2}\), a więc ze zbioru o zerowej mierze, stąd \(\displaystyle{ P(\lim_{n \to \infty}
\frac{S_{n}}{n}\neq p) = 0}\)
(mamy zbiór miary zero) oraz \(\displaystyle{ P(X \in B) = 1}\) ( \(\displaystyle{ X}\) przyjmuje te wartości ze zbioru o zerowej mierze z prawdopodobieństwem \(\displaystyle{ 1}\)), gdzie \(\displaystyle{ B = \mathbb{R} \setminus A}\).
A więc istnieje taki zbiór.

Czy to jest całe rozwiązanie tego zadania i pokazanie, że ta zmienna losowa ma rozkład osobliwy, czy czegoś tam brakuje?

Bardzo proszę o wszelką pomoc!!!
Ostatnio zmieniony 26 kwie 2015, o 16:38 przez Jan Kraszewski, łącznie zmieniany 1 raz.
Powód: Poprawa wiadomości i tematu.
ODPOWIEDZ